Proof that canonical transformation implies symplectic condition

In summary: Goldstein's Classical Mechanics makes the claim (pages 382 to 383) that given coordinates ##q,p##, Hamiltonian ##H##, and new coordinates ##Q(q,p),P(q,p)##, there exists a transformed Hamiltonian ##K## such that ##\dot Q_i = \frac{\partial K}{ \partial P_i}## and ##\dot P_i = -\frac{\partial K}{Q_i}## if and only if ##MJM^T= J## where ##M## is the Jacobian of ##Q,P## with respect to ##q,p## and ##J = \begin{bmatrix}O&I
  • #1
Lagrange fanboy
9
2
TL;DR Summary
Goldstein's classical mechanics shows the proof that if symplectic condition holds, then the transformation is canonical. The converse was claimed to be true, but I can't derive it.
Goldstein's Classical Mechanics makes the claim (pages 382 to 383) that given coordinates ##q,p##, Hamiltonian ##H##, and new coordinates ##Q(q,p),P(q,p)##, there exists a transformed Hamiltonian ##K## such that ##\dot Q_i = \frac{\partial K}{ \partial P_i}## and ##\dot P_i = -\frac{\partial K}{Q_i}## if and only if ##MJM^T= J## where ##M## is the Jacobian of ##Q,P## with respect to ##q,p## and ##J = \begin{bmatrix}
O&I\\\\
-I&O
\end{bmatrix}##. I understood the book's proof that ##MJM^T= J## implies the existence of such ##K##. However, the proof of the converse was not given and I do not know how to derive it myself.
 
  • Like
Likes PhDeezNutz
Physics news on Phys.org
  • #2
A Hamiltonian function is not changed under the canonical transformation independent on time: K=H
 
  • Like
Likes topsquark
  • #3
Lagrange fanboy said:
TL;DR Summary: Goldstein's classical mechanics shows the proof that if symplectic condition holds, then the transformation is canonical. The converse was claimed to be true, but I can't derive it.

Goldstein's Classical Mechanics makes the claim (pages 382 to 383) that given coordinates ##q,p##, Hamiltonian ##H##, and new coordinates ##Q(q,p),P(q,p)##, there exists a transformed Hamiltonian ##K## such that ##\dot Q_i = \frac{\partial K}{ \partial P_i}## and ##\dot P_i = -\frac{\partial K}{Q_i}## if and only if ##MJM^T= J## where ##M## is the Jacobian of ##Q,P## with respect to ##q,p## and ##J = \begin{bmatrix}
O&I\\\\
-I&O
\end{bmatrix}##. I understood the book's proof that ##MJM^T= J## implies the existence of such ##K##. However, the proof of the converse was not given and I do not know how to derive it myself.
This proposition is actually wrong. Indeed, the transformation P=p, Q=2q does not satisfy ##MJM^T= J##. But after this transformation a Hamiltonian system remains a Hamiltonian one with the new Hamiltonian K=2H.
The Hamiltonian formalism is a subtle enough mathematical topic to study it by physics textbooks
 
Last edited:
  • #4
wrobel said:
A Hamiltonian function is not changed under the canonical transformation independent on time: K=H
No, usually you have an additional term. The most convenient way is to use a generating function, e.g., ##f(q,Q,t)##. Then the canonical transformation is given as
$$p=\partial_q f, \quad P=-\partial_Q f, \quad H'=H+\partial_t f.$$
 
  • #5
Please read the post you are quoting :)
 
  • Like
Likes weirdoguy
  • #6
By the way: Most general definition of a canonical transformation of an extended phase space is as follows.
A transformation
$$(t,x,p)\mapsto(T,X,P)$$ is said to be canonical if
$$dp_i\wedge dx^i-dH\wedge dt=dP_i\wedge dX^i-dK\wedge dT,$$ where
##H=H(t,x,p),\quad K=K(T,X,P).## Such transformations take a Hamiltonian system
$$\frac{dp}{dt}=-\frac{\partial H}{\partial x},\quad \frac{dx}{dt}=\frac{\partial H}{\partial p}$$
to a Hamiltonian one
$$\frac{dP}{dT}=-\frac{\partial K}{\partial X},\quad \frac{dX}{dT}=\frac{\partial K}{\partial P}.$$
But the most common type of canonical transformations is a special case of the above:
$$(t,x,p)\mapsto(T,X,P),\quad T=t.\qquad (*)$$
Introduce a notation
$$f=f(t,x,p),\quad \delta f=\frac{\partial f}{\partial x^i}dx^i+\frac{\partial f}{\partial p_i}dp_i.$$
Theorem. A transformation (*) is canonical iff ##\delta P_i\wedge\delta X^i=\delta p_i\wedge\delta x^i.##

Here we assume that independent variables are ##p,x,t## that is
$$P=P(t,x,p),\quad X=X(t,x,p)\qquad (**)$$ and correspondingly ##dx=\delta x,\quad dp=\delta p.##

If the transformation (**) is canonical and ##X=X(x,p),\quad P=P(x,p)## then
$$H(t,x,p)=K(t,X(x,p),P(x,p))$$
 
Last edited:
  • Like
Likes vanhees71
  • #7
Excuse the Juvenile comment but the first time I heard my classical mechanics teacher say "Kamiltonian" I struggled to withhold my laughter. I couldn't believe he said it with a straight face.
 
  • #8
PhDeezNutz said:
Excuse the Juvenile comment but the first time I heard my classical mechanics teacher say "Kamiltonian" I struggled to withhold my laughter. I couldn't believe he said it with a straight face.
In Russian, the letter H in the word Hamiltonian is pronounced as g in the word gone. Oh, I see for English speakers this Kamiltonian sounds somehow like camel
 
Last edited:
  • Haha
Likes vanhees71 and PhDeezNutz

Related to Proof that canonical transformation implies symplectic condition

What is a canonical transformation?

A canonical transformation is a change of coordinates in phase space that preserves the form of Hamilton's equations. It transforms the old coordinates and momenta into new coordinates and momenta while maintaining the structure of the Hamiltonian system.

What is the symplectic condition?

The symplectic condition refers to a property of a transformation that preserves the symplectic structure of phase space. Mathematically, it means that the Jacobian matrix of the transformation satisfies a specific condition involving the symplectic matrix, ensuring that the transformation preserves the canonical commutation relations.

How does a canonical transformation imply the symplectic condition?

A canonical transformation implies the symplectic condition because, by definition, it preserves the Hamiltonian structure of the system. This preservation is mathematically equivalent to the transformation being symplectic, meaning the Jacobian of the transformation satisfies the symplectic condition.

Can you provide a mathematical expression for the symplectic condition?

Yes, the symplectic condition can be expressed as \( J^T \Omega J = \Omega \), where \( J \) is the Jacobian matrix of the transformation, and \( \Omega \) is the symplectic matrix, typically represented as a block matrix with identity and negative identity blocks.

Why is the symplectic condition important in Hamiltonian mechanics?

The symplectic condition is crucial because it ensures that the essential properties of the Hamiltonian system, such as conservation laws and phase space volume, are preserved under the transformation. This preservation is vital for the physical consistency and predictability of the system's evolution.

Similar threads

Replies
3
Views
909
Replies
3
Views
298
  • Advanced Physics Homework Help
Replies
1
Views
802
Replies
4
Views
755
  • Mechanics
Replies
5
Views
1K
  • Advanced Physics Homework Help
Replies
9
Views
1K
Replies
19
Views
2K
  • Advanced Physics Homework Help
Replies
3
Views
927
Replies
3
Views
672
Replies
5
Views
615
Back
Top